LSAT and Law School Admissions Forum

Get expert LSAT preparation and law school admissions advice from PowerScore Test Preparation.

 Administrator
PowerScore Staff
  • PowerScore Staff
  • Posts: 8916
  • Joined: Feb 02, 2011
|
#40710
Complete Question Explanation
(The complete setup for this game can be found here: lsat/viewtopic.php?t=3641)

The correct answer choice is (E)

If J is third, then Q cannot be third, and from the contrapositive of the second rule Q :longline: S. Because Q :longline: S, and from the third rule S :longline: Y, we know that a Q :longline: S :longline: Y is in effect.

However, because Q and S are both nutrition articles, they cannot be consecutive, so even if Q is first, S can never be second (again, this was previously known, but if you missed that inference, here is an alternate way to arrive at it for the purposes of this question). Since S cannot be second, answer choice (D) can never occur and is thus incorrect.

Aside from the Q :longline: S :longline: Y chain, the other chain in effect is the J :longline: G :longline: R chain created by the fourth rule. This chain also has limitations created by the first rule, namely that J and G cannot consecutive. Because J is already third, consequently G cannot be fourth, and answer choice (A) can be eliminated.

The remaining two incorrect answer choices are harder to eliminate, but all revolve around violations of the first rule. Let’s look at each:

Answer choice (B): If H, the random, is sixth, then G must be fifth and R must be seventh. However, this forces Q and S into first and second, respectively (and Y into fourth), and Q and S cannot be consecutive as they are both nutrition articles.

Answer choice (C): If Q is first, then the earliest S can be is fourth. Thus, among the last four articles are S :longline: Y, and G :longline: R. While those four articles can co-exist among the last four, the remaining article, H, is forced into second. This creates a violation because H and J—which are both finance articles—are then consecutive.

Accordingly, only answer choice (E) remains in contention. Y can be fifth in this hypothetical solution: H-Q-J-S-Y-G-R. Thus, answer choice (E) could be true and is correct.

Get the most out of your LSAT Prep Plus subscription.

Analyze and track your performance with our Testing and Analytics Package.